چند جمله ای صحیح

چند جمله ای صحیح

فرض کنید a,b, و c  سه عدد صحیح متمایز، و  P  یک چند جمله ای با ضرایب عدد صحیح. نشان دهید که غیر ممکن است که P(a) = b, P(b) = c, و P(c) = a باشد.


a,b,c \in \mathbb{Z} و a,b,c متمایز است. 
از آنجایی که ما P[\mathbb{Z}] ، فرض کنید ما با شرایط داده شده به exisitng به طوری که 
a-b | b-c 
b-c | c-a 
c-a | a-b است. 
همانطور که ما در برخورد بیش از اعداد صحیح، 
2b\ge c+a 
2c\ge a+b 
2a\ge b+c 
با این حال نشان میدهد equaltiy  در همه جا و در نتیجه تناقض است.


فرض کنید که
 a < b < c است. سپس |P(a) - P(c)| = |b-a|< |a-c| ، یک تناقض است. 
بعد، فرض کنید که b < a < c است. سپس |P(b) - P(c)| = |c - a| < |c-b| ، همچنین یک تناقض است. 

توجه داشته باشید که این هر دو تناقضات هستند چون |a-c| تقسیم |P(a) - P(c)| و |c-b| تقسیم |P(b) - P(c)| است.


با استفاده از این واقعیت شناخته شده است که
 (a-b)\mid P(a)-P(b) ما دریافت می کنید 

  • \frac{P(a)-P(b)}{a-b}=\frac{b-c}{a-b}=k_1

 

  • \frac{P(b)-P(c)}{b-c}=\frac{c-a}{b-c}=k_2

 

  • \frac{P(c)-P(a)}{c-a}=\frac{a-b}{c-a}=k_3


که در آن k_i جدایی ناپذیر است. 

ضرب معادلات اول و آخر ما\left(\frac{P(a)-P(b)}{a-b}\right)\left(\frac{P(c)-P(a)}{c-a}\right)=\frac{b-c}{c-a}=k_1k_3.اما \frac{b-c}{c-a}=\frac{1}{k_2} ، بنابراین تنها راه برای k_2 به یک عدد صحیح است اگر |k_2|=1 است. به همین ترتیب ما دریافت کنید که |k_1|=|k_3|=1 است. اگر یکی از k_1,k_2,k_3 -1 است، WLOG می گویند k_1 ، و سپس \tfrac{b-c}{a-b}=-1\iff{b-c=-a+b}\iff{a=c} ، تناقض است. بنابراین، k_1,k_2,k_3 همه برابر با 1، و متصل کردن این در آن شرح زیر است که 2a=b-c ، 2b=c-a و 2c=a-b است. حل این سیستم عملکرد a=b=c=0 ، اما a,b,c اعداد صحیح متمایز تعریف، یک تناقض است.

 


شما می توانید نتیجه را بلافاصله با استفاده از این واقعیت  که :
اگر x|y و y|x ، و سپس |x|=|y| است.

بدست آورید...!

 

نویسنده مطلب: Meysam Zarei

Meysam Zarei

پاسخ دهید

هیچ نظری تا کنون برای این مطلب ارسال نشده است، اولین نفر باشید...